The words you are searching are inside this book. To get more targeted content, please make full-text search by clicking here.

هذا الكتاب يشمل خاص بالمبياد الرياضيات للدول العربية يشمل الاطوار الثلاثة .متوسط وثانوي وجامعي .يحتوي على ازيد من 500 تمرين مع الحل بطرق جديدة

Discover the best professional documents and content resources in AnyFlip Document Base.
Search
Published by Zeraoulia Rafik, 2023-04-20 00:39:56

المبياد الرياضيات للدول العربية

هذا الكتاب يشمل خاص بالمبياد الرياضيات للدول العربية يشمل الاطوار الثلاثة .متوسط وثانوي وجامعي .يحتوي على ازيد من 500 تمرين مع الحل بطرق جديدة

Keywords: IMO

i الحل .f(1) = 1 يكون ان يجب هنا. 1 ≤ f(n) ≤ 2n − 1 لدينا ومنه f(f(n)) = 2n − f(n) ≥ 1 ان نلاحظ من السهل التاكد من ان f تطبيق غامر نفرض ان k) = k(f من اجل n, ..., 1 = k .نلاحظ انه يجب ان يكون لدينا 1+n) ≥ 1+n(f مادام f تطبيق ,f(f(n+1))+f(n+1) = 2(n+1) ومادام, f(f(n+1)) ≥ n+1 الـكيفية بنفس لدينا يكون ان يجب غامر .f(n + 1) = n + 1 يكون ان يجب 8مثال 99 جد جميع الدوال بحيث f(m2 + f(n)) = f(m) 2 + n من اجل جميع الاعداد الطبيعية (1, 2, 3, 4, ..) i الحل ان نلاحظ. f(m2 + f(n)) = f(m) 2 + n على تدل P(m, n) ليكن P(1, y) =⇒ f(1 + f(y)) = f(1)2 + y =⇒ P(f(x), 1 + f(y)) =⇒ f(f(x) 2 + f(1)2 + y) = f(f(x))2 + 1 + f(y) (1) P(x, y) =⇒ f(x 2 + f(y)) = f(x) 2 + y P(f(1), x2 + f(y)) =⇒ f(f(x) 2 + f(1)2 + y) = f(f(1))2 + x 2 + f(y) (2) (1),(2) =⇒ f(f(x))2 = x 2 + c x مربع تام لاي عدد صحيح موجب x ,ومنه 0 = c) بخلاف ذلك يمكن اختيار 2 +c ,هنا. c = f(f(1))2 −1 مع x ,والذي يقود نحو التناقض ). 2 + c < x2 + x < (x + 1)2 بحيث x > c وبالتالي. f(f(x)) = x ,هنا P(1, f(x)) =⇒ f(x + 1) = f(x) + f(1)2 ومنه f(x) = f(1)2x + (f(1) − f(1)2 ) . مادام x = f(f(x)) = f(1)4x + f(1)2 (f(1) − f(1)2 ) + (f(1) − f(1)2 ) يجب ان يكون لدينا f(1) = 1 .f(x) = x ومنه, 201 201


9 مثال 1010 حل المعادلة الدالية الاتية f(x) = f ( 1 x ) , x > 0 i الحل نعتبر اي دالة معرفة بالشكل الاتي g : [1, +∞) → R حيث g(1) = 2 . R : f المعرفة ب ∗ ومنه الدالة R+ → f(x) = g(1/x) كان اذا 0 < x < 1 و f(x) = g(x) كان اذا x ≥ 1 هي حل المعادلة الدالية السابقة 10 مثال 1111 ليكن Z مجموعة الاعداد الصحيحة . حدد جميع الدوال Z → Z : f بحيث من اجل كل الاعداد الصحيحة a و b ,لدينا f(2a) + 2f(b) = f(f(a + b)). i الحل نلاحظ ان f(f(x)) = f(2x) + 2f(0) و f(2x) = 2f(x) − f(0) ومنه f(2a) + f(2b) = f(2a + 2b) + f(0) وبالتالي 2(f(a + 1) − f(a)) = f(2) − f(0) وبالتالي f(2) − f(0) = 2m ومنه f(a + 1) − f(a) = m بالتراجع ينتج f(a) = m(a − 1) + c 202 202


مع (0(f = c .نقوم بالتعو يض في شروط المشكل نحصل على : m(2a − 1) + c + 2(c + m(b − 1)) = f(m(a + b − 1) + c) = mc + m2 (a + b − 1) + c , والتي هي مكافئة للمعادلة الاتية m2 (s − 1) − m(2s − 2) + c(m − 2) = 0 m = 2 وعليه منتهية ليست s لـكن s − 1|c(m − 2) او f(x) = 0 ومنه c = 0 ومنه m = 0 كان اذا. s = a + b مع (0 = c ,اعلاه) ومنه مجموعة الحلول هي الدوال الخطية f(x) = 2x + c مع C عدد صحيح 11 مثال 1212 f(x + y) = f(x) + f(y) بحيث f : R → R الدوال جميع جد i الحل f(x + y) = f(x) + f(x) f(0) = 0 f(−x) = −f(x) كل هذا كافي لنحكم عن كل هذه الدوال عبارة عن دالة خطية دون اي ثابت اي , f ′ (x) = f(x + h) − f(x) h اذا كان f ′ (x) = f(h) h = c ومنه الحل الوحيد هو cx) = x(f 12 مثال 1313 جد جميع الدوال N→N : f التي تحقق f(m2 + n 2 ) = f(m) 2 + f(n) 2 , ∀m, n ∈ N . N = {1, 2, 3, . . .} , هنا 203 203


i الحل الحل الوحيد هو f(n) = n . الجزء الصعب هو تحديد وايجاد 1) = 1(f .ليكن a) = 1(f .العبارة المعطاة تطبق على اي عدد والذي هو مجموع مربعين وبالتالي f(2) = f(1 + 1) = 2a 2 f(5) = f(4 + 1) = (2a 2 ) 2 + a 2 = 4a 4 + a 2 f(8) = f(4 + 4) = 8a 4 f(10) = f(9 + 1) = f(3)2 + a 2 f(13) = f(9 + 4) = f(3)2 + 4a 4 بالرغم ان بعض الاعداد هي عبارة عن مجموع مربعين باكثر من طر يقة ولـكن نعتبر هذا الطر يق مفتاح البرهان 7 2 + 1 = 52 + 52 , f(7)2 = 2f(5)2 − a 2 2 2 + 112 = 52 + 102 , f(11)2 = f(10)2 + f(5)2 − f(2)2 112 + 72 = 132 + 1, f(11)2 = f(13)2 − f(7)2 + a 2 5 2 + 142 = 102 + 112 , f(14)2 = f(10)2 + f(11)2 − f(5)2 6 2 + 132 = 32 + 142 , f(6)2 = f(14)2 + f(3)2 − f(13)2 المعادلات الثلاثة الاولى تعطي لنا هاته المتغيرات لنا تعطي f(7)2 مع f(11)2 باختزال. f(11)2 ,f(7)2 ,f(3)2 a 2 (4a 2 − 1)f(3)2 = a 2 (2a 2 + 1)2 (4a 2 − 1) .f(3) = 2a 2 + 1 so ومنه, f(14)2 يمكن استعمال المعادلتين الاخيرتين لايجاد f(6)2 = 2(1 + 8a 2 + 17a 4 + 8a 6 − 16a 8 ) هذا يجᣞبرنا على ان تكون 1 = a مادام كثير الحدود ياخذ قيم سالبة من اجل 1 > a .فانه ينتج مباشرة n) = n(f من اجل القيم الصغيرة ل n ,بالتعو يض , وهنا بالتراجع 1414 اسئلة اختبار ية A 1 ليكن N مجموعة الاعداد الصحيحة الموجبة . جد جميع الدوال N → N : f حيث ان k | f(m + n) ⇐⇒ k | f(m) + f(n) من اجل كل الاعداد الصحيحة الموجبة k. 2 جد جميع الدوال f : Z≥0 → Z≥0 f(f(m) + f(n)) = m + n. بحيث 204 204


R مع ∗ ∗ → R ∗ R : f ∀n ∈ N f(n+1)>f(f(n)), : الاتي الشرط تحقق التي f : N → N الدوال جميع جد 3 :y ̸= −x 2 R مع ∗ مجموعة الاعداد الحقيقية غير المعدومة . جد جميع الدوال التي تحقق المعادلة الدالية الاتية x و y في f ( x 2 + y ) = f(x) 2 + f(xy) f(x) 54 جد جميع الدوال C → Q : f التي تحقق ,x1, x2, . . . , x1988 ∈ Q كل اجل من) 1 • f(x1 + x2 + · · · + x1988) = f(x1)f(x2). . . f(x1988) . (2 • f(1988)f(x) = f(1988)f(x) . من اجل كل Q ∈ x. 6 جد جميع الدوال R → R : f حيث من اجل كل عدد حقيقي x و y, f ( y 2 − f(x) ) = yf(x) 2 + f ( x 2 y + y ) . حيث f : Q+ → Q+ الدالة لتكن 7 f(x) + f ( 1 x ) = 1 و f(2x) = 2f ( f(x) ) . جد ببرهان الصيغة الضمنية لعبارة الدالة (x(f من اجل جميع قيم العدد الحقيقي الموجب x. 8 حدد جميع الدوال R → R : f بحيث f ( x 2 y + f(y) ) = x 2 f(y) + f ( f(y) ) من اجل كل R ∈ y, x. 9 حدد حلول المعادلة الدالية الاتية f(x) + f(x + y) = y + 2 10 حدد جميع الدوال R → R : f التي تحقق المعادلة الدالية الاتية f ( x + yf(x) ) = f(x)f(y) , من اجل جميع الاعداد الحقيقية x و y. 1515 اسئلة اختبار ية B 1 جد (x(f من اجل R → R : f ,التي تحقق الشرط الاتي f(x + y 3 ) = f(x) + [f(y)]3 .f ′ (0) 0 و x, y ∈ R كل اجل من. 205 205


2 نفرض ان الدالة R → R : f قابلة للاشتقاق و f ′ ( x + y 2 ) = f(x) − f(y) x − y محققة من اجل y ̸= x . .حل المعادلة الدالية السابقة من اجل f. h مع حل f(y) = h(log |y|) ان وجدنا. f(y) + f ( 1 y ) = 0 بحيث ثابة غير دالة f : R \ {0} → R لتكن 3 دالة فردية . هل توجد حلول اخرى 4 حل المعادلة الدالية الاتية f(x + y) − f(x − y) = 2f ′ (x)f ′ (y) من اجل كل R → R : f غير ثابتة وقابلة للاشتقاق 5 جد جميع الدوال Z → Z : f بحيث من اجل كل الاعداد الصحيحة Z ∈ m, n ,لدينا f(n 2 + m) = f(n + m2 ). XXIV حلول تمارين لالمبيادات دولية سابقة TST Sur Cono .Argentina 2014 11 حدد جميع الثنائيات الموجبة للاعداد الاولية (q, p (حيث ان p 5 + p 3 + 2 = q 2 − q i الحل مادام (q − 2, q + 1) = 1, 3 فان q − 2 < q + 1 ايضا ومادام p فان p لايساوي 3 يقسم احد هاته الاعداد 1 + q, 2 − q . بالضبط نفسه مع 3 p 3 = q + 1 p 2 + 1 = q − 2 p ومنه الحل الوحيد هو 2 = p و 7 = q 3 − p 2 − 4 = 0 اي Contest AIME,USA 2017 22 جد عدد الاعداد الصحيحة الموجبة n اقل من 2017 بحيث 1 + n + n 2 2! + n 3 3! + n 4 4! + n 5 5! + n 6 6! عدد صحيح 206 206


i الحل n عدد صحيح . نحصل على مقام مشترك , لدينا 2 2! + n 3 3! + n 4 4! + n 5 5! + n 6 نلاحظ انه يكفي ان نجعل !6 360n 2 + 120n 3 + 30n 4 + 6n 5 + n 6 720 n عدد صحيح الان نلاحظ ان 2 حيث انها تستلزم ان (2 mod (0 ≡ n .وبالتالي , !2 n 3 3! + n 4 4! + n 5 5! + n 6 6! = 120n 3 + 30n 4 + 6n 5 + n 6 720 . n عدد صحيح . ختاما نحن 3 3! + n 4 4! , وبالتالي. n ≡ 0 (mod 3) ومنه n 6 ≡ 0 (mod 3) ان نلاحظ, mod 3 باخذ متاكدين بان n 5 5! + n 6 6! Z ∈ k .الان لدينا 6n . الان لدينا 6k = n من اجل + 5+n 6 عدد صحيح . وهذا يساوي 720 6n 5 + n 6 720 = 6 6 (k 5 + k 6 ) 720 . وبالتالي يكفي ايجاد k بحيث k 5 + k 6 ≡ 0 (mod 5) ومنه k ≡ 0, 1 (mod 5) ان نلاحظ. n ≡ 6, 0 (mod 30) . باستعمال بعض الحسابات القاعدية نلاحظ انه يوجد 134 = 67 · 2 من اجل كل الاعداد الصحيحة الموجبة n الاقل من 2017 AMC12.AHSME2021 33 يحتوي مخروطان دائريان قائمان برؤوس متجهة لأسفل كما هو موضح في الشكل أدناه على نفس الكمية من السائل. يبلغ نصف قطر قمم الأسطح السائلة 3 سم و 6 سم. في كل مخروط يتم إسقاط قطعة رخامية كرو ية نصف قطرها 1 سم ، والتي تغرق في القاع وتغمر بالكامل دون أن ينسكب أي سائل. ما هي نسبة ارتفاع مستوى السائل في المخروط الضيق إلى ارتفاع مستوى السائل في المخروط العر يض؟ شكل 6.3 :الشكل هل هي (A) 1 : 1 (B) 47 : 43 (C) 2 : 1 (D) 40 : 13 (E) 4 : 1 207 207


i الحل نعلم ان حجم المخروط هو 1 3 πr2h. نعلم ان نصف قطر المخروط الاول هو 3 ونصف قطر المخروط الثاني هو .6 لذلك نضع النسبة 1 3 π9h : 1 3 π36h. بتبسيط هاته النسبة لدينا 3hπ : 12hπ 1 : 4. إذا تم إسقاط قطعة من الرخام ، سيرتفع كلا المخروطين وفقًا للنسبة 1 : 4. لذا فان الاجابة هي (E( A1 IMO Shortlist 2016 44 ان برهن. min(ab, bc, ca) ≥ 1 بحيث موجبة حقيقية اعداد c ,b ,a لتكن √3 (a 2 + 1)(b 2 + 1)(c 2 + 1) ≤ ( a + b + c 3 )2 + 1. 208 208


i الحل لاثبات صحة المتراجحة السابقة نحاول ان نبرهن صحة هذه الخاصية . فان xy ≥ 1 و x, y > 0 كان اذا: (x 2 + 1)(y 2 + 1) ≤ (( x + y 2 )2 + 1)2 . خاصية برهان بطرح الطرفين نحصل على 1 16 (x − y) 2 ( x 2 + 6xy + y 2 − 8 ) بحيث انها ليست سالبة ■ عند هذه النقطة نفرض ان c ≥ 1 ≥ b ≥ a ,وهذا يتركنا نفرض ان b = a ,بسلاسة a .xy ≥ 1 و x ≥ 1 ≥ y مع) a, b, c) = (x, x, y) النتيجة هاته صحة نبرهن ان يكفي وبالتالي . والمتقاربان b و (y − x (يمكن الحصول على الصعب بهاته الـكيفية 2 ويمكن ان نقوم بهذا مباشرة بالطرح وايضا نقوم بتحليل العبارة , = d فنحصل على a+b+c 3 بحيث نترك (a 2 + 1)(b 2 + 1)(c 2 + 1)(d 2 + 1) ≤ (( a + d 2 )2 + 1)2 (( b + c 2 )2 + 1)2 ≤ (( a + b + c + d 4 )2 + 1)4 = ( d 2 + 1)4 ■ (.a = b = c = 1 لما فقط وليس, a = b = c ≥ 1 لما المساواة تكون( Math Brazilian Olympiad 2005 55 برهن ان ∑∞ n=1 1 nn = ∫ 1 0 x −x dx. i الحل ∫ 1 0 x −x dx = ∑∞ n=0 ∫ 1 0 (−x ln x) n n! 209 209


∫ 1 0 x n (ln x) n dx = 1 n + 1 ∫ 1 0 (ln x) n dx n+1 = 1 n + 1 ∫ 1 0 (ln x 1 n+1 ) n dx = 1 (n + 1)n+1 ∫ 1 0 (ln x) n dx, وبالتراجع لدينا , ∫ 1 0 (ln x) n dx = x(ln x) n | 1 0 − n ∫ 1 0 (ln x) n−1 = 0 − n(n − 1)!(−1)n−1 = (−1)nn!, ∫ 1 0 x −x dx = ∑ 1 ومنه nn Russian 2019 MO 66 برهن a + b + c 4 ≥ √ ab − 1 b + c + √ bc − 1 c + a + √ ca − 1 a + b . i الحل في البداية تبدو المتراجحة ليست بالجميلة ولـكن بعد النظر الجيد اصبحت جد جذابة ورائعة ليكن 1 + x = a ,1 + y = b, ,c = z + 1 .x, y, z ⩾ 0 ان نلاحظ في البداية نحاول ان نكتب المتراجحة السابقة على الشكل الاتي x + y + z + 3 2 ⩾ 2 √ xy + x + y y + z + 2 + 2 √ yz + y + z x + z + 2 + 2 √ xz + x + z x + y + 2 . بتطبيق متراجحة ,GM-AM نحصل على x + y + 2 = (x + 1) + (y + 1) ⩾ 2 √ (x + 1)(y + 1) ⇐⇒ 1 √ (x + 1)(y + 1) ⩾ 2 x + y + 2 . نقوم بشكل مماثل للثنائيات (1 + z, 1 + x (و (1 + z, 1 + y (ونحصل على المتراجحة المراد برهانها او اثباتها x + y + z + 3 2 ⩾ √ xy + x + y (y + 1)(z + 1) + √ yz + y + z (x + 1)(z + 1) + √ xz + x + z (x + 1)(y + 1). الان بتطبيق متراجحة ,Schwarz-Cauchy نحصل على (∑ cyc xy + x + y y + 1 ) (∑ cyc 1 x + 1) ⩾ (√ xy + x + y (y + 1)(z + 1) + √ yz + y + z (x + 1)(z + 1) + √ xz + x + z (x + 1)(y + 1))2 . نلاحظ ان ∑ cyc xy + x + y y + 1 = x + y + z + ∑ cyc x x + 1 = x + y + z + 3 − ∑ cyc 1 x + 1 . = s .لذلك نحن بحاجة الى تبيان ان 1 x+1 + 1 y+1 + 1 z+1 و t = x + y + z + 3 بوضع الان ( t 2 )2 ⩾ (t − s)s ⇐⇒ t 2 − 4ts + 4s 2 ⩾ 0 ⇐⇒ (t − 2s) 2 ⩾ 0. 210 210


3rd-2019 round MO Iran 77 b, a و c اعداد حقيقية موجبة اي ان ∑ cyc (a + b) 2 = 2∑ cyc a + 6abc . برهن ان ∑ cyc (a − b) 2 ≤ 2 ∑ cyc a − 6abc . i الحل (∑ cyc (a − b) 2 )2 ≤ (∑ cyc (a + b) 2 − 12abc)2 (∑ cyc (a − b) 2 )2 ≤ (∑ cyc (a + b) 2 )2 + 144a 2 b 2 c 2 − 24abc (∑ cyc (a + b) 2 ) 3abc (∑ cyc (a + b) 2 − 6abc) ≤ (∑ cyc (a 2 + b 2 ) ) (∑ cyc ab) , 3abc ( 2 ∑ cyc a ) ≤ (∑ cyc (a 2 + b 2 ) ) (∑ cyc ab) 3abc(a + b + c) ≤ (a 2 + b 2 + c 2 )(ab + ac + bc) IMO 2021 88 برهن ان ∑n i=1 ∑n j=1 √ |xi − xj | ⩽ ∑n i=1 ∑n j=1 √ |xi + xj | محققة من اجل جميع الاعداد الحقيقية .xn, . . . x1 211 211


i الحل اذا كان 0 = xi من اجل بعض القيم ل i ,يقود نحو 0 = xn .ومنه بالتراجع لدينا (1−xn, . . . , x2, x1,( ∑ 1≤i,j≤n √ |xi + xj | = 2 · ∑ 1≤i≤n−1 √ |xi | + ∑ 1≤i,j≤n−1 √ |xi + xj | ≥ 2 · ∑ 1≤i≤n−1 √ |xi | + ∑ 1≤i,j≤n−1 √ |xi − xj | = ∑ 1≤i,j≤n √ |xi − xj | √ من اجل كل |xi + xj | + √ |xi − xj وبخلاف ذلك يمكن ان نفرض ان 0 ̸= xi من اجل كل i ,وبالتالي 0| > نكتب وايضا. x0 = 0 مع, yi = |xi | − |xi−1| نعرف وكذلك|, x1| ≤ |x2| ≤ |x3| ≤ · · · ≤ |xn| ومنه, i, j εi = xi من اجل كل i. |xi | ∑ i ∑ j √ |xi + xj | − √ |xi − xj | = ∑ i ∑ j |xi + xj | − |xi − xj | √ |xi + xj | + √ xi − xj | = 2∑ i ∑ j εiεj min(|xi |, |xj |) √ |xi + xj | + √ |xi − xj | ≥ 2 maxi,j √ |xi + xj | + √ |xi − xj | (∑ i ∑ j εiεj min(|xi |, |xj |) ) ≥ 2 maxi,j √ |xi + xj | + √ |xi − xj |   ∑ i ∑ j εiεj ∑ k≤min(i,j) yk   = 2 maxi,j √ |xi + xj | + √ |xi − xj | (∑ k yk ∑ i,j≥k εiεj ) = 2 maxi,j √ |xi + xj | + √ |xi − xj |   ∑ k yk (∑ i≥k εi )2   ≥ 0 IMO2020 99 لتكن الاعداد الحقيقية d, c, b, a بحيث a ≥ b ≥ c ≥ d > 0 .a + b + c + d = 1 برهن ان (a + 2b + 3c + 4d)a a b b c c d d < 1 212 212


i الحل يمكن بسهولة اثباتها باستعمال متراجحة GM-AM تسمى في بعض الـكتب بمتراجحة الوزن وعليه لدينا a 2 + b 2 + c 2 + d 2 ≥ a a b b c c d d بحيث نصل الى , (a + 2b + 3c + 4d)a a b b c c d d ≤ (a + 2b + 3c + 4d)(a 2 + b 2 + c 2 + d 2 ) االان اذا كان هذه المتراجحة تحقق [ 1 2 ≤ a < 1 ] فان (a + 2b + 3c + 4d)a a b b c c d d ≤ 1 − 2(1 − a) 2 (2a − 1) والتي هي من الواضح اقل من 1 ختاما اذا كانت هاته المتراجحة تحقق مايلي [ 0 < a < 1 2 ] فان (a + 2b + 3c + 4d)a a b b c c d d ≤ 1 − (1 − a)(1 − 2a) < 1 والتي تبدو وضوحا انها اقل ايضا من 1 والذي يبرهن صحة المشكل المعطى IMO2015 1010 ليكن ABC مثلث حاد مع AC > AB .ليكن Γ محيطها H مركزها العمود مع F عند سفح الارتفاع من A .ليكن M نقطة المنتصف ل BC .ليكن Q نقطة تقع على محيط المثلث Γ بحيث ◦90 = HQA ∠ولنفرض ان K النقطة على المحيط Γ بحيث ◦90 = HKQ .∠نفرض ان هاته النقاط A ,B ,C ,K و Q متمايزة وتقع على Γ بهذا الترتيب . برهن ان دوائر المثلثات KQH و FKM مماسة لبعضها البعض i الحل خاصية01 النقاط M, H, Q متداخلة خطيا او مرتبطة خطيا . 213 213


برهان مادام BHCA1 متوازي اضلاع فان , A1, M, H ايضا متداخلة خطيا .من ∠AQH = ∠AQA1 = 90◦ , فالخاصية محققة . □ ■ خاصية02 النقطة T هي مركز محيط المثلث ALA0. برهان نلاحظ ان A0 T = A T وكذلك مادامت النقاط F, H, Q, T مرتبطة خطيا نحصل على ∠AT F = ∠MHF = ∠A0HQ′ = ∠A0LQ′ = 180◦ − ∠ALA0, ومنه الخاصية محققة . □ ■ ′K, Q مرتبطة خطيا , A′ خاصية03 برهان بقوة النقطة , T Q · T A = T B · T C = TK′ · T L, ′AQ′ ∥ QA ,فان النتيجة صحيحة . □ مادام وكذلك QK′ ∥ AL وبالتالي. T Q = TK′ ■ ومنه نلاحظ ختاما ان ∠T QH = ∠QK′H = ∠HK′A ′ = 90◦ =⇒ T Q مماس)QK′H). ′TK مماس للدائرة (H′QK .(نطبق العكس عند M ذات نصف القطر ′TK = Q T ينتج ان المستقيم من هاته المساواة MC = MB ولاحظ ان الثنائيات او الازواج الاتية {ω, Γ}, {Q, H}, {T, F}, {K, K′ } 214 214


ٺتبادل فيما بينها . انطلاقا من ′TK مماس (H′QK (نحصل على (KQH (و (KFM( مماسين لبعضهما البعض وهو المطلوب IMO2013 1111 نفرض ان k و n اعداد صحيحة موجبة .برهن انه توجد اعداد صحيحة موجبة mk, . . . , m1 بحيث 1 + 2 k − 1 n = ( 1 + 1 m1 ) · · · ( 1 + 1 mk ) . i الحل يمكن ان نبرهن صحة القضية بالتراجع من اجل k. الشرط الابتدائي : 1 = k .اذن فقط نختار n = m1 .مع n عدد فردي .اي ان 1 + 2 k − 1 n = 2 k + (n − 1) n = n + 1 n · 2 k + (n − 1) n + 1 = ( 1 + 1 n ) (1 + 2 k − 2 n + 1 ) = ( 1 + 1 n ) (1 + 2 k−1 − 1 (n + 1)/2) . باستعمال فرضية التراجع يوجد 1−mk, . . . , m1 بحيث 1 + 2 k−1 − 1 (n + 1)/2 = ( 1 + 1 m1 ) · · · ( 1 + 1 mk−1 ) , ومنه بترك n = mk نحصل على صحة القضية المراد اثباتها . الحالة الثانية n عدد زوجي . ليكن 2m = n .ومنه 1 + 2 k − 1 n = 2 k + (2m − 1) 2m = 2 k + (2m − 1) 2 k + (2m − 2) · 2 k + (2m − 2) 2m = ( 1 + 1 2 k + (2m − 2)) (2 k−1 + (m − 1) m ) = ( 1 + 1 2 k + (2m − 2)) (1 + 2 k−1 − 1 m ) . باستعمال فرضية التراجع يوجد 1−mk, . . . , m1 بحيث 1 + 2 k−1 − 1 m = ( 1 + 1 m1 ) · · · ( 1 + 1 mk−1 ) , ومنه بوضع (2 − 2m + (2k = mk نكون قد برهنا صحة القضية المراد اثباتها 215 215


IMO2014 1212 ليكن . . . a2 < a1 < a0 متتالية غير منتهية من الاعداد الصحيحة الموجبة . برهن انه يوجد عدد صحيح وحيد 1 ≥ n بحيث an < a0 + a1 + a2 + · · · + an n ≤ an+1. i الحل Let an = a0 + ∆1 + ∆2 + ... + ∆n من اجل كل 1 ≥ n .نلاحظ ان 1 ≥ n∆ > an يمكن صياغتها بالشكل الاتي a0+...+an n ≤ an+1 الشروط , الان a0 + ∆1 + ∆2 + ... + ∆n < (n + 1)a0 + n∆1 + (n − 1)∆2 + ... + ∆n n ≤ ≤ a0 + ∆1 + ... + ∆n + ∆n+1 بعد بعض الحسابات يمكن الحصول على ∆2 + 2∆3 + ... + (n − 1)∆n < a0 ≤ ∆2 + 2∆3 + ... + (n − 1)∆n + n∆n+1 الان نعرف f : N → Z و f(1) = 0 ب f(n) = ∆2 + 2∆3 + .. + (n − 1)∆n نبرهن ان يبقى الان). f(n + 1) − f(n) = n∆n+1 ≥ n) محدودة وغير متزايدة f ان نلاحظ, n ≥ 2 اجل من, تماما متزايدة f و f(1) = 0 < a0 مادام واضح وهذا f(n) < a0 < f(n + 1) ان بحيث n ∈ N وحيد عدد يوجد انه ومحدودة وبالتالي تم حل المشكل 1313 الصين الدورة الثانية 2021 في المثلث ABC ,△النقطة M منتصف AC .حيث AB//MD يلتقي بمماس A الى (ABC ⊙(عند D .النقطة E تقع على AD والنقطة A منتصف القطعة المستقيمة DE .مع {P} = ⊙(ABE) ∩ AC, {Q} = ⊙(ADP) ∩ DM . برهن ان ∠QCB = ∠BAC . . يمكن الاستعانة بالشكل ادناه 216 216


i الحل فقط سنحاول استعمال هاته العلاقة الهندسية (1) AD = AE =⇒ Q ∈ (△ACE) مطاردة. AC منتصف M و MN ∥ AB لان BC منتصف N ومنه, MD ∥ AB ان نعلم. MD ∩BC = N ليكن الزاو ية تؤدي الى ان يكون ADCN دوري اي ∠CAD = ∠ABC = ∠MNC = ∠DNC ايضا∠ DQP = ∠P AD = ∠CAD = ∠DNC =⇒ P Q ∥ BC الان باستعمال (1 (نعلم ان تقاطع المماس الى (ABC (عند C و QP يقع على (ADCN .(لتكن هذه النقطة هي F. الان يمكن مطاردة الزاو ية ∠ACF = ∠CBA = ∠CAD =⇒ AC ∥ F D لان ADF C دوري وكذلك∠ ACF منحرف شبه وهو D ACF =⇒ CAD= ∠ ∠F CM = ∠F CA = ∠CBA = ∠CNM = ∠P QM = ومنه, ND ∥ AB وكذلك P Q ∥ BC لدينا وايضا ∠F QM =⇒ F CQM دوري ND ∥ AB =⇒ ∠BAC = ∠AMD = ∠FMC F CQMcyclic = ∠F QC P Q∥BC = ∠QCB =⇒ ∠BAC = ∠QCB 217 217


يمكن الاستعانة بالشكل الاتي والذي يلخص حوصلة الحل 1414 الصين الدورة الثانية 2014 ان برهن . abc > 0 و a + b + c = 1 بحيث حقيقية اعداد a, b, c ليكن bc + ca + ab < √ abc 2 + 1 4 . i الحل √ abc 2 = √ abc(a + b + c) 2 = 1 4 ( (2∑bc − ∑a 2 )2 + 1 3 ( 5 ∑a 2 − 4 ∑bc)(2 ∑bc − ∑a 2 ) + 1 9 ( 2a 2 − b 2 − c 2 + 2bc − ca − ab)2 ) 1 2 ≥ 1 4 ( 2 ∑bc − ∑a 2 ) = ∑bc − 1 4 218 218


1515 الصين الدورة الثانية 2012 .x, y, z ∈ [0, 1] ان نفرض جد القيمة العظمى للعبارة √ |x − y| + √ |y − z| + √ |z − x|. i الحل : وبالتالي x = max(x, y, z) ان نفرض √ |y − z| + √ |z − x| + √ |x − y| = √ x − y + √ x − z + √ |y − z|. اذا كان z ≥ y وبالتالي : √ x − y + √ x − z + √ |y − z| = √ x − y + √ x − z + √ y − z ≤ √ (1 + 1)(x − y + y − z) + √ x − z = (√ 2 + 1)√ x − z ≤ 1 + √ 2 في حالة y ≥ z لدينا : √ x − y + √ x − z + √ |y − z| = √ x − y + √ x − z + √ z − y ≤ √ (1 + 1)(x − z + z − y) + √ x − y = (√ 2 + 1)√ x − y ≤ 1 + √ 2 x, y, z ل ية ودور x = 1, y = 0, 5, z = 0 لما محققة 1 + √ x, y, z ل ية ودور x = 1, y = 0, 5, z = 0 لما محققة 1 + 2 العظمى القيمة√ القيمة العظمى 2 ملاحظة ربما نستعمل طر يقة اخرى للبرهان نستحضر فيها الدوال المثلثية والمتراجحات المثلثية على هذا النحو . ليكن √ |y − z| = a, √ |z − x| = b باستعمال المتراجحة المثلثية لدينا a 2 + b 2 = |y − z| + |z − x| ≥ |y − x|. وبالتالي ∴ √ |y − z| + √ |z − x| + √ |x − y| ≤ a + b + √ a 2 + b 2 مع |a| ≤ 1, |b| ≤ 1 219 219


, وبالتالي توجد بعض الاعداد الحقيقية θ بحيث a = cos θ, b = sin θ (0 ≤ θ < 2π) a + b + √ a 2 + b 2 = cos θ + sin θ + 1 = √ 2 cos ( θ − π 4 ) + 1 ≤ 1 + √ 2. √ = b = a ,والتي تحقق 1 2 = θ ,تؤدي وتقود الى π 4 √ + 1 لما وبالتالي القيمة الاعظمية او الحدية 2 |a| ≤ 1, |b| ≤ 1. المتوسط للتعليم 2017-2018 SDML دورة 1616 احسب مايلي . 3 4 + 37 84 . .(A) 27 (B) 29 (C) 33 (D) 37 (E) 39 i الحل لدينا 3 4 + 37 84 = 3 4 (1 + 33 ) 3(28) = 3 4 (28) 3(28) = 33 = 27 وبالتالي الاجابة الصحيحة هي A التعليم المتوسط -السلاسل الحسابية دورة 2017 -1717 2018 اذا كان (x + 1) + (x + 2) + ... + (x + 20) = 174 + 176 + 178 + ... + 192 (A) 80 (B) 81 (C) 82 (D) 83 (E) 84 هي هل? x اقيمة فماهي, i الحل 20x + 210 = 915 =⇒ 10x = 810 =⇒ x = 81 220 220


1818 التعليم المتوسط2017 5 من مساحة الدائرة في الرسم التخطيطي ادناه دائرتان مركزهما D ونصف قطريهما 1 و 2 .اجمالي مساحة المنطقة المظللة هو 12 الاكبر كم درجة عند قياس الزاو ية ADC . ?∠هل هي (A) 100 (B) 105 (C) 110 (D) 115 (E) 120 i الحل ليكن x كسرا ل 360 درجة .قياس ADC هو اذن 3πx + (1 − x)π 4π = 5 12 =⇒ 1 + 2x = 5 3 =⇒ x = 1 3 1 3 اذن الاجابة الصحيحة هي 120) E = (360∗ school SMDL2017.M 1919 كم عدد الطرق المتاحة لتغطية هذه المنطقة بالدومينو؟ 221 221


i الحل تأخذ الدومينو مربعين لكل منهما. يمكننا فصل الرسم البياني أعلاه إلى أربع مربعات كل منها تسع وحدات مع مربع أسود واحد في المنتصف. من خلال التجربة ، يتضح أن الدومينو لا يمكن أن يكون في مربعين ؛ خلاف ذلك ، سيكون للمربع عدد فردي من الوحدات ، حيث سيكون من المستحيل وضع قطعة الدومينو. هناك طر يقتان إجمالتان لوضع أربع قطع من 2 طر يقة للتغطية الدومينو في مربع واحد ، وهو ما يعطي 16 = 4 2020 الالمبياد الوطني لتركيا2020 الاصاغر جد جميع الاعداد الحقيقية للثنائيات (y, x (التي تحقق 2x 2 + y 2 + 7 = 2(x + 1)(y + 1) i الحل عند نشر العبارة السابقة نحصل على 2x 2 + y 2 + 7 = 2(x + 1)(y + 1) ولنكتب العبارة الاخيرة على شكل مربع من اجل x ,نحصل على 2x 2 − (2y + 2)x + y 2 − 2y + 5 222 222


مادام x عدد حقيقي معناه مميزه او محدده سيكون 0 ,≥ وهذا معناه (2y + 2)2 − (4)(2)(y 2 − 2y + 5) ≥ 0 =⇒ (y + 1)2 − 2(y 2 − 2y + 5) ≥ 0 =⇒ (y − 3)2 ≤ 0 والذي يحدث ويتحقق فقط لما 3 = y ,نعوض في المعادلة نحصل على x 2 − 4x + 4 = 0 =⇒ (x − 2)2 = 0 =⇒ x = 2 اذن الحل الوحيد هو (x, y) = (2, 3) APMO2002 2121 جد جميع الاعداد الصحيحة الموجبة a و b بحيث a 2 + b b 2 − a و b 2 + a a 2 − b عددين صحيحين 223 223


i الحل نعلم ان : a 2 + b ≥ b 2 − a ⇐⇒ a 2 + a ≥ b 2 − b ⇐⇒ ( a + 1 2 )2 ≥ ( b − 1 2 )2 ⇐⇒ a + 1 2 ≥ b − 1 2 ⇐⇒ a − b ≥ −1b 2 + a ≥ a 2 − b ⇐⇒ b 2 + b ≥ a 2 − a ⇐⇒ ( b + 1 2 )2 ≥ ( a − 1 2 )2 ⇐⇒ b + 1 2 ≥ a − 1 2 ⇐⇒ a − b ≤ 1 فان a = b + 1 كان اذا) 1 a 2 + b b 2 − a = b 2 + 3b + 1 b 2 − b − 1 = 1 + 4b + 2 b 2 − b − 1 نلاحظ انه لما 7 ≥ b ,1 − b − b2 < 2 + 4b ,فان مانقوم به يدو يا هو التحقق من قيم b من 0 الى 6 ,والتي تعطي لنا الحلول الاتية (a, b) = (1, 2),(2, 3) . 2 (اذا كان b = a فان a 2 + b b 2 − a = a 2 + a a 2 − a = a + 1 a − 1 = 1 + 2 a − 1 وهذه تعطي لنا (a, b) = (2, 2),(3, 3) b = a + 1 (3 b 2 + a a 2 − b = a 2 + 3a + 1 a 2 − a − 1 = 1 + 4a + 2 a 2 − a − 1 والتي تعطي لنا بنفس الحالة الاولى الحلول الاتية (a, b) = (2, 1),(3, 2) 224 224


2021 JMO USA 2222 ليكن N ترمز الى مجموعة الاعداد الصحيحة الموجبة . جد جميع الدوال N → N : f بحيث من اجل العددين الصحيحين الموجبين a و ,b لدينا f(a 2 + b 2 ) = f(a)f(b) and f(a 2 ) = f(a) 2 . i الحل الحل الوحيد هو , 1 ≡ f والذي من الواضح انه يحقق المعادلة . لنبرهن ان 1) = n(f من اجل جميع قيم العدد الطبيعي ,n نستخدم التراجع من اجل .n الحالة: القاعدية 1 = n 2)1(f) = 1(f ومنه مادام (1(f عدد طبيعي اذن يجب ان يكون .1 بوضع 1 = a في القضية الثانية يقود نحو , : مرحلة التراجع 1 (n عدد فردي n, n) = (b, a (في القضية الاولى نحصل على 2−1 2 نضع ( f ( n 4 + 2n 2 + 1 4 ) = f(n)f ( n 2 − 1 2 ) . n القضية الثانية تصبح 4+2n 2+1 4 = (n 2+1 2 ) 2 مادام , f ( n 2 + 1 2 )2 = f(n)f ( n 2 − 1 2 ) . 1−n) = (b, a (في القضية الاولى نحصل على 2 , n+1 2 ومن جهة اخرى بتعو يض ( f ( n 2 + 1 2 ) = f ( n − 1 2 ) f ( n + 1 2 ) , والتي تساوي 1 باستعمال فرضية التراجع . f(n) = 1. بجعل f(n)f ( n 2−1 2 ) وبالتالي ,1= n ≥ 4 زوجي عدد n (2 n, n) = (b, a (في القضية الاولى تعطي لنا 2−4 4 بوضع ( f ( n 4 + 8n 2 + 16 4 ) = f(n)f ( n 2 − 4 4 ) . n القضية الثانية تقود نحو المعادلة الاتية 4+8n 2+16 16 = (n 2+4 4 ) 2 مادام , f ( n 2 + 4 4 )2 = f(n)f ( n 2 − 4 4 ) . 225 225


n) = (b, a (في القضية الاولى نحصل على 2 ومن جهة اخرى بوضع (1, f ( n 2 + 4 4 ) = f (n 2 ) f(1), والتي تساوي 1 انطلاقا من فرضية التراجع هنا f(n)f( n 2 − 4 4 ) = 1, ومنه (n(f يجب ان تكون .1 IMO2010 2323 جد جميع الدوال R → R : f بحيث من اجل كل R ∈ y, x المعادلة الاتية محققة f(⌊x⌋ y) = f(x) ⌊f(y)⌋ مع ⌊a ⌊اكبر عدد صحيح ليس اكبر من .a i الحل من اجل : 0 = x نحصل على : f(0) = f(0) ⌊f(y)⌋ اذن اما f(0) = 0 او ⌊f(y)⌋ = 1 من اجل كل y و 1)⌋ = y(f ⌊عبارة ايضا عن حل من اجل كل y. : على نحصل x = n ∈ N ∗ الان اذا كان 0) = 0(f : بوضع 1 = y و f(n) = f(n) ⌊f(1)⌋ اذا كان 0) = n(f من اجل كل n ومنه من اجل n = x يجب ان يكون لدينا 0) = yn(f من اجل كل y ومنه f(y) = 0 والذي ايضا نعتبره حل . : x = n اجل من وكذلك⌋ f(1)⌋ = 1 و f(0) = 0 كان اذا f(ny) = f(n) ⌊f(y)⌋ 226 226


من f(n) = f(n + p) : اما معناه f(ny) = f(n + p) ⌊f(y)⌋ :p ∈ (0, 1) و n ∈ Z مع x = n + p اجل من لـكن y كل اجل من⌋ f(y)⌋ = 0 او p ∈ (0, 1) اجل ⌊f(1)⌋ = 1 ومنه : f(n) = f(n + p)forp ∈ (0, 1) f(n + 1) = f(⌊n⌋ n+1 n ) = f(n) ⌊ f(1 + 1 n ) ⌋ = f(n). ⌊f(0)⌋ = 0 : لدينا n < 0 حيث صحيح عدد n ليكن ومنه 0) = x(f من اجل كل 2 ≤ −x الان ليكن n عدد صحيح 1 > n : لدينا : f(n + 1) = f(⌊n⌋ n + 1 n ) = f(n) ⌊ f(1 + 1 n ) ⌋ = f(n). ⌊f(1)⌋ = f(n) = c f(n) = f(n + p) باستعمال c = 0 على نحصل y = −1 و x = −2 اجل من x ≥ 2 كل اجل من f(x) = c ومنه من اجل (1, 0 ∈ (p سيكون لدينا :    f(x) = 0 , x ∈ (−∞, −1) ∪ [0, 1) ∪ (2, +∞) f(x) = f(−1) , x ∈ [−1, 0) f(x) = f(1) , x ∈ [1, 2) .2(f) = 1(f والتي تبرز لنا تناقضا مع الفرض اذن الحلول هي 1 2 ) = f(2). ⌊ f( 1 2 ) ⌋ لـكن من جهة اخرى لدينا : 0= f(x) = 0 f(x) = c, c ∈ [1, 2[ ICM2019 2424 ليكن R) → 1, 1 : (−f قابلة للاشتقاق مرتين بحيث 2f’(x) + xf′′(x) ⩾ 1 for x ∈ (−1, 1). . برهن ان ∫ 1 −1 xf(x)dx ⩾ 1 3 . 227 227


i الحل لدينا 2f ′′(x) + xf′ (x) ≥ 1 الحالة الاولى 0 > x بالتكامل نجد =⇒ ∫ x 0 2f ′ (t)dt + ∫ x 0 tf′′(t)dt ≥ 1 باستعمال التكامل بالتجزئة نحصل على 2 (f(x) − f(0)) + xf′ (x) − f(x) + f(0) ≥ x =⇒ f(x) − f(0) + xf′ (x) ≥ x x .∫ عند تبسيط التكامل بالتجزئة السابق نحصل على 0 f(t)dt + ∫ x 0 tf′ (t) − f(0)x ≥ x 2 2 . بالتكامل ايضا نجد xf(x) ≥ x 2 2 + f(0)x . نضع 1 = x للحصول على ∫ 1 0 xf(x)dx ≥ ∫ 1 0 ( t 2 2 + f(0)t)dt = 1 6 + f(0) 2 (1) ومنه. f(x) − f(0) + xf′ الحالة الثانية 0 < x نستعمل نفس المنهجية السابقة في الحالة الاولى نحصثل على x) ≤ x( ∫ 0 −1 xf(x)dx ≥ 1 6 − f(0) 2 (2) نجمع المتراجحتين السابقتين طرف الى طرف نجد اي (2) + (1 (نحصل على ∫ 1 −1 xf(x)dx ≥ 1 3 IMO 2000Shortlist 2525 ليكن c, b, a اعداد حقيقية موجبة حيث 1 = abc . .برهن ان ( a - 1 + 1 b ) (b - 1 + 1 c ) (c - 1 + 1 a ) ≤ 1. 228 228


i الحل لدينا ∆ = ∏ cyc ( a - 1 + 1 b ) = ∏ cyc (a - abc + ac) = abc∏ cyc (1 - bc + c) = ∏ cyc ( 1 - 1 a + c ) = ∏ cyc ( a + 1 - 1 b ) ⇒ ∆ 2 = ∏ cyc ( a - 1 + 1 b ) (a + 1 - 1 b ) = ∏ cyc [ a 2 - ( 1 - 1 b )2 ] ≤ a 2 b 2 c 2 = 1 ⇒ ( a - 1 + 1 b ) (b - 1 + 1 c ) (c - 1 + 1 a ) ≤ 1 a, b, c, 1 a , 1 b , 1 c ويمكن استعمال طر يق اخر بتوظيف نظر ية Pigeon Hole على النحو الاتي مادام 1 = abc ثلاث اعداد حقيقية موجبة ) تكون موجبة باستعمال نظر ية Pigeon Hole. a - 1 + 1 b ) , ( b - 1 + 1 c ) , ( c - 1 + 1 a ) ومنه على الاقل اثنين من اذا كانت احدى القيم السابقة سالبة فان المتراجحة واضحة لان ( a - 1 + 1 b ) (b - 1 + 1 c ) (c - 1 + 1 a ) ≤ 0 ≤ 1. لذلك نحن بحاجة لان تكون كل القيم الثلاثة السابقة موجبة . باستعمال متراجحة GM-AM و 1 = abc فان 6 = ( a - 1 b + 1) + ( b - 1 c + 1) + ( c - 1 a + 1) + ( 1 a - c + 1) + ( 1 b - a + 1) + ( 1 c - b + 1) ≥ 6 6 √( a - 1 b + 1) (b - 1 c + 1) (c - 1 a + 1) (1 a - c + 1) (1 b - a + 1) (1 c - b + 1) = 6√6 a (1 - c + bc) b (1 - a + ca) c (1 - b + ab) c (b - 1 + ab) a (c - 1 + bc) b (a - 1 + ca) = 6√6 ca (ab - 1 + b) ab (bc - 1 + c) bc (ca - 1 + a) c (b - 1 + ab) a (c - 1 + bc) b (a - 1 + ca) 229 229


= 6 6 √ (abc) 3 ( a - 1 + 1 b )2 ( b - 1 + 1 c )2 ( c - 1 + 1 a )2 = 6 3 √( a - 1 + 1 b ) (b - 1 + 1 c ) (c - 1 + 1 a ) وبالتالي , 1 ≥ 3 √( a - 1 + 1 b ) (b - 1 + 1 c ) (c - 1 + 1 a ) نقوم بتكعيب الطرفين نحصل على ( a - 1 + 1 b ) (b - 1 + 1 c ) (c - 1 + 1 a ) ≤ 1. , ( a - 1 b + 1) = ca ( 1 c - 1 + b ) , ( 1 a - c + 1) = c ( 1 c - 1 + b ) نلاحظ انه مادام ca و c عددين موجبين مع ) موجبة . 1 a - c + 1) و ( a - 1 b + 1) فانه يمكننا تدويرها بالـكيفية الاتية بحيث ( a - 1 b + 1) , ( b - 1 c + 1) , ( c - 1 a + 1) , ( 1 a - c + 1) , ( 1 b - a + 1) , ( 1 c - b + 1) اعداد حقيقية موجبة المبياد الر ياضيات المفتوح Petersburg Saint 2626 2021 ليكن c, b, a اعداد مركبة برهن ان | a 2 ab + ac − bc| + | b 2 ba + bc − ac | + | c 2 ca + cb − ab| ≥ 3 2 مع ان المقامات ليست معدومة i الحل نضع x = a 2 bc − ab − ac y = b 2 ac − ba − bcz = c 2 ab − ca − cb ثم نقوم بتعو يضها في الطرف الايسر للمتراجحة السابقة ولنتاكد بعدها من صحة المساواة الاتية xy + xz + yz + 2xyz = 1. > |z| + |y| + |x |فانه باستعمال متوسط المتراجحات نحصل على 3 2 ومنه اذا كان |xy + xz + yz| ≤ |x| · |y| + |x| · |z| + |y| · |z| < 3 4 في حين |xyz| < 1 8 وهذا تناقض 230 230


Baltic 2020way 2727 ليكن 0 > a0 عدد حقيقي ولتكن المتتالية an = √ an−1 1 + 2020 · a 2 n−1 , for n = 1, 2, . . . , 2020. a2020 < 1 . بين ان 2020 i الحل لاحظ ان 1 a 2 n = 1 a 2 n−1 + 2020 =⇒ 1 a 2 n − 1 a 2 n−1 = 2020. باخذ المجموع التلسكوبي والذي يعني اختزال الحدود المتعاقبة جمعا بالتناوب فانه يمكن الحصول على 1 a 2 n = 1 a 2 1 + 2020(n − 1). الان a1 = a0 1 + 2020 · a 2 0 < 1 √ 2020 =⇒ 1 a 2 1 > 2020. هنا , 1 a 2 n > 2020n =⇒ an < 1 √ 2020n . وبالتالي النتيجة او المطلوب Batlic way 2016 2828 جد جميع الثنائبات للاعداد الاولية (q, p (بحيث p 3 − q 5 = (p + q) 2 . i الحل لدينا (p + q) 2 = p 3 − q 5 q ≥ p =⇒ 0 > p3 − q 5 = (p + q) 2 ومنه p > q (p + q) 2 ≡ p 2 (mod q) وكذلك p 3 − q 5 ≡ p 3 (mod q) =⇒ p 3 ≡ p 2 (mod q) =⇒ q|p 3 − p 2 = p 2 (p − 1) =⇒ q|p − 1 231 231


(p + q) 2 ≡ q 2 (mod p) وايضا p 3 − q 5 ≡ −q 5 (mod p) =⇒ q 2 ≡ −q 5 (mod p) =⇒ p|q 2 + q 5 = q 2 (q 3 + 1) =⇒ p|q 3 + 1 p|(q + 1)(q 2 − q + 1) =⇒ p|q + 1 او p|q 2 − q + 1 نميز حالتين (i p|q + 1 =⇒ q + 1 ≥ p و q|p − 1 =⇒ p − 1 ≥ q =⇒ p − 1 ≥ q ≥ p − 1 =⇒ p − 1 = q =⇒ q = 2 3 لايوجد حل 3 − 2 5 < 0 لـكن p = 3 وكذلك (ii p|q 2 − q + 1 q|p − 1 =⇒ p − 1 = qk, p = qk + 1 و qk + 1|q 2 − q + 1 =⇒ q 2 − q + 1 = (qk + 1)t = qkt + t x ≥ 1 =⇒qkt + t ≡ t (mod q) =⇒ t ≡ 1 (mod q) =⇒ t = qx + 1q 2 − q + 1 ≡ 1 (mod q) x = 0 =⇒ t =qkt + t ≥ q(q + 1)k + q + 1 ≥ q(q + 1) + q + 1 = q 2 + 2q + 1 > q2 − q + 1 =⇒ p, q >1 =⇒ qkt + t = qk + 1 = q 2 − q + 1, qk = q 2 − q =⇒ k = q − 1 =⇒ p = q 2 − q + 1 q ≡ 2 (mod 3) =⇒ q 2 − q + 1 ≡ p ≡ 0 (mod 3) 3 =⇒ p, q ≡ 1, 2 (mod 3) ومنه q ≡ 1 (mod 3) =⇒ q 2 − q + 1 ≡ 1 ≡ p (mod 3) =⇒ p, q ≡ 1 (mod 3) =⇒ p 3 − q 5 ≡ 0 ≡ (p + q) 2 ≡ 2 2 ≡ 1 (mod 3) ومنه لايوجد حل =⇒ p = 3 او q = 3 p = 3 =⇒ q = 2(p > q) اي لايوجد حل . q = 3 =⇒ p = 7 ومنه (p, q) = (7, 3) 232 232


Batlic way 2018 2929 جد كل الدوال (∞,0, ∞) → [0 : [f ,بحيث من اجل كل عدد صحيح موجب n ومن اجل اي عدد حقيقي ليس سالب بحيث x1, x2, . . . , xn f(x 2 1 + . . . + x 2 n ) = f(x1) 2 + · · · + f(xn) 2 . i الحل لدينا f(a 2 ) = f(a) 2 وهذا معناه f(a 2 + 2ab + b 2 ) = f( √ a 2 + b 2 ) 2 + f( √ 2ab) 2 = f(a 2 + b 2 ) + f(2ab) من اجل 0 ≥ y ≥ x يمكن ان نجد دوما b, a بحيث x = a 2 + b 2 , y = 2ab, a = √ x + y + √ x − y 2 , b = √ x + y − √ x − y 2 ومنه f(x + y) = f(x) + f(y) → f(x) = kx اي f(a) 2 = f(a) 2 → ka2 = k 2 a 2 → f(x) = 0, x APMO2018 3030 لتكن (x(f و (x(g دالتان عدديتان معرفتان بالشكل الاتي f(x) = 1 x + 1 x − 2 + 1 x − 4 + · · · + 1 x − 2018 g(x) = 1 x − 1 + 1 x − 3 + 1 x − 5 + · · · + 1 x − 2017 . برهن ان |f(x) − g(x)| > 2 من اجل اي عدد حقيقي ليس بعدد صحيح x يحقق 2018 < x < 0 i الحل .Ij = (j, j + 1) الاتي المجال نعرف, j = 0, 1, 2, · · · , 2017 كل اجل من نلاحظ انه من اجل كل 1 < α < 0, 1 α(α + 1) + 1 (1 − α)(2 − α) = 2n + 2 n(n + 2) > 2 .0 < n = α − α 2 < 1 4 مادام الان نميز اربع حالات . 233 233


x = 2n−1+α وليكن, 1 ≤ n ≤ 1008 حيث القيم بعض اجل من x ∈ I2n−1 ليكن x ∈ ∪1008 j=1 I2j−1 الاولى الحالة .0 < α < 1 مع | f(x) − g(x) |= | 1 x − 1 x − 1 + 1 x − 2 − · · · − 1 x − 2017 − 1 2018 − x | =| 1 x(x − 1) + 1 (x − 2)(x − 3) + · · · + 1 (x − 2016)(x − 2017) + 1 2018 − x | ≥ 1 (x − 2n + 2)(x − 2n + 1) + 1 (x − 2n)(x − 2n − 1) = 1 α(α + 1) + 1 (1 − α)(2 − α) > 2 x ∈ ∪1008 j=1 I2j الحالة الثانية : ومنه. 0 < α < 1 بعض اجل من x = 2n + α وليكن, 1 ≤ n ≤ 1008 بعض اجل من x ∈ I2n ليكن | f(x) − g(x) |=| 1 x − 1 x − 1 + 1 x − 2 − · · · + 1 x − 2017 − 1 2018 − x | =| 1 x + 1 (x − 1)(x − 2) + 1 (x − 3)(x − 4) + · · · 1 (x − 2017)(x − 2018) | ≥ 1 (x − 2n + 1)(x − 2n) + 1 (x − 2n − 1)(x − 2n − 2) = 1 α(α + 1) + 1 (1 − α)(2 − α) > 2. الحالة الثالثة : I0 ∈ x | f(x) − g(x) |=| 1 x − 1 x − 1 + 1 x − 2 − · · · + 1 x − 2017 − 1 2018 − x | =| 1 x(x − 1) + 1 (x − 2)(x − 3) + · · · + 1 (x − 2016)(x − 2017) + 1 2018 − x | ≥ 1 x + 1 (x − 1)(x − 2) ≥ 1 x(x + 1) + 1 (x − 1)(x − 2) > 2 الحالة الرابعة : I2017 ∈ x ومنه. 0 < α < 1 مع x = 2017 + α ليكن | f(x) − g(x) |=| 1 x − 1 x − 1 + 1 x − 2 − · · · − 1 x − 2017 − 1 2018 − x | =| 1 x + 1 (x − 1)(x − 2) + 1 (x − 3)(x − 4) + · · · 1 (x − 2017)(x − 2018) | ≥ 1 (x − 2015)(x − 2016) + 1 (x − 2017)(x − 2018) ≥ 1 α(α + 1) + 1 (1 − α)(2 − α) > 2. ,x ∈ ∪2017 هنا من اجل كل Ij 0=j |f(x) − g(x)| > 2 , كما هو مطلوب 234 234


APMO2002 3131 ليكن z, y, x اعداد موجبة بحيث 1 x + 1 y + 1 z = 1. برهن ان √ x + yz + √ y + zx + √ z + xy ≥ √ xyz + √ x + √ y + √ z i الحل لدينا ∑√ x + yz = ∑√ x xyz xy + yz + zx + yz = ∑√ yz xy + yz + zx (x 2 + xy + yz + zx) = ∑ √ yz(x + y)(x + z) xy + yz + zx = 1 √ xy + yz + zx ∑√ yz(x + y)(x + z) ≥ 1 √ xy + yz + zx ∑√ yz(x + √ yz) 2 = 1 √ xy + yz + zx ∑(x √ yz + yz) = ∑( √ x 2yz xy + yz + zx + yz √ xy + yz + zx ) = ∑√ x + ∑yz √ xy + yz + zx = ∑√ x + √ xy + yz + zx = ∑√ x + √ xyz 3232 الدورة النهائية كور يا 2011 جد القيمة العظمى او الحدية للعبارة الجᣞبرية الاتية ,اذا كان c, b, a اعداد موجبة بحيث 1 = c + b + a. 1 a 2 − 4a + 9 + 1 b 2 − 4b + 9 + 1 c 2 − 4c + 9 235 235


i الحل مادام 1 a 2 − 4a + 9 ≤ a + 2 18 فان 1 a 2 − 4a + 9 + 1 b 2 − 4b + 9 + 1 c 2 − 4c + 9 ≤ a + b + c + 6 18 = 7 18 المساواة تتحق لما (a, b, c) = (1, 0, 0),(0, 1, 0),(0, 0, 1) 3333 حل في مجموعة الاعداد الحقيقية الجملة الاتية { (x − 1)(y − 1)(z − 1) = xyz − 1, (x − 2)(y − 2)(z − 2) = xyz − 2. i الحل ومنه. z = c + 1 ,y = b + 1 ,x = a + 1 الاتي يض بالتعو نقوم (a + 1)(b + 1)(c + 1) = abc + 1, (a + 1)(b + 1)(c + 1) = (a − 1)(b − 1)(c − 1) + 2. كحالة خاصة (a − 1)(b − 1)(c − 1) = abc − 1. هاته المعادلات تعطي (ab + bc + ca) + (a + b + c) = 0 −(ab + bc + ca) + (a + b + c) = 0 and a 2 + b 2 + c 2 = (a + b + c) 2 − 2(ab + bc + ca) = 0 وبالتالي. a + b + c = ab + bc + ca = 0. ومنه . x = y = z = 1 تستلزم, a = b = c = 0 ومنه كحل x = y = z = 1 بملاحظة مدعم بملاحظة مدعم) x, y, z x, y, z) = ( a + 1 , b + 1 , c + 1) يض التعو يض التعو ملاحظة 236 236


3434 الالمبياد الوطني للاصاغر تركيا 2012 ليكن y, x اعداد صحيحة مع p عدد اولي من اجل x 2 − 3xy + p 2 y 2 = 12p جد جميع الثلاثيات (p, y, x( i الحل نعيد كتابة المعادلة السابقة مع جعل الطرف الايمن معدوم فنحصل على : x 2 − 3xy + p 2 y 2 − 12p = 0 من الواضح ان مميز المعادلة المعطى بالشكل الاتي 9y 2 − 4p 2 y 2 + 48p = d 2 مربع تام . اذا كان 3 ̸= p فان p 2 ≡ 1(mod3) و 9y 2 − 4p 2 y 2 + 48p ≡ −4p 2 y 2 ≡ 2y 2 ≡ d 2 (mod3) مادامت الرواسب التربيعية (mod3 (هي 1 و 0 ,يمكن ان نصل الى 3|y y = 3y1 ⇒ x 2 − 9xy1 + 9p 2 y 2 1 = 12p ⇒ 3|x ⇒ x = 3x1 9x 2 1 − 27x1y1 + 9p 2 y 2 1 = 12p 3x 2 1 − 9x1y1 + 3p 2 y 2 1 = 4p 3|LHS اي ان 3 لايقسم الطرف الايسر وبالتالي تناقض ⇒ p = 3 237 237


⇒ x = 3x1 ⇒ 3|x x2 − 3xy + 9y 2 = 36 x 2 1 − x1y + y 2 = 4 قمنا بحل المعادلة التربيعية السابقة بجعلها صفر ية مع جعل مميزها مربع تام وحصلنا على الحلول الاتية (p, x, y) = (3, 6, 0),(3, 6, 2),(3, 0, 2),(3, 0, −2),(3, −6, −2) 3535 المبياد البراز يل مستوى جامعي 2018 بحيث دالتين f, g : R → R ليكن f(x + g(y)) = −x + y + 1 من اجل كل ثنائية للاعداد الحقيقية x و y .ماهي قيمة g(x + f(y) ? i الحل ليكن (y, x(P تدل على التاكيد الاتي , f(x + g(y)) = −x + y + 1 . P (x − g(x + f(y)), x + f(y)) =⇒ g(x + f(y)) = f(x) − f(y) − 1 P(x − g(y), y) =⇒ f(x) + x = g(y) + y + 1 =⇒ f(x) = −x + C مع R ∈ C .وبالتالي , g(x + f(y)) = −x + C + y − C − 1 = −x + y − 1 Austrian-Polish2005 3636 ليكن ABCD رباعي محدب مع CD = AB .ارسم المثلثين ABE و CDF خارج الرباعي المحدب ABCD اي ان DCF = ∠ABE ∠و .∠BAE = ∠FDC برهن ان منتصفات القطع المستقيمة الاتية AD ,BC و EF مرتبطة خطيا i الحل نفرض ان منتصفات القطع المستقيمة الاتية EF, AD و BC هي Z, Y, X على الترتيب . نقوم بالانسحاب على D و F F .ومنه يكون وضوحا ′ ′D و F الى بواسطة الشعاع ⃗CB الذي ياخذ D الى F F⃗ ′ = DD⃗ ′ = CB⃗ . انطلاقا من منتصفات اجزاء المثلثات لدينا XX⃗ ′ = Y Y⃗ ′ = 1/2CB⃗ = ZB⃗ ′BF′D ,∆فان Z, Y, X ايضا مرتبطة خطيا وهو المطلوب ′X مرتبطة خطيا بالتناظر بين ABE ∆و , Y ′ . مادام B, 238 238


Austrian-Polish2006 3737 جد كل كثيرات الحدود (x(P بمعاملات حقيقية الذي يحقق المعادلة الاتية (x + 1)3P(x − 1) − (x − 1)3P(x + 1) = 4(x 2 − 1)P(x) من اجل كل الاعداد الحقيقية x i الحل يكون. P(x) = xQ(x) بوضع اذن, x | P(x) هنا, 8P(0) = 0 على نحصل x = 1 اجل من (x + 1)3 (x − 1)Q(x − 1) − (x − 1)3 (x + 1)Q(x + 1) = 4(x 2 − 1)xQ(x) او (x + 1)2Q(x − 1) − (x − 1)2Q(x + 1) = 4xQ(x) نكتب المعادلة الاخيرة على الشكل الاتي : (x 2 + 1)(Q(x − 1) − Q(x + 1)) = 2x(2Q(x) − Q(x − 1) − Q(x + 1)) فان, deg Q(x) = n ⩾ 2 كان اذا• deg(Q(x − 1) − Q(x + 1)) = n − 1 و deg(2Q(x) − Q(x − 1) − Q(x + 1)) = n − 2 , هنا الطرف الايسر للمعادلة الاخيرة من الدرجة 1 + n والطرف الايمن لها من الدرجة 1 − n .وهذا تناقض . 239 239


• اذا كان Q(x) = ax + c , فانه بعد الترتيب نحصل على −2a(x 2 + 1) = 0 =⇒ a = 0 . وبالتالي Q(x) = c, c ∈ R و P(x) = cx VTRMC1979 3838 بين ان حجم الاسطوانة الدائرية V والتي لها اقل مساحة سطح هي الاسطوانة التي يساوي قطرها ارتفاعها (الجزء العلوي والسفلي جزء من السطح). i الحل ليكن r و h نصف القطر والارتفاع للاسطوانة على الترتيب . لدينا V = πr2h و SA = 2πr2 + 2πrh باستعمال متراجحة ,GM-AM لدينا SA = 2πr2 + πrh + πrh ≥ 3 √3 2π 3r 4h 2 = 3√3 2πV 2 نحصل على المساواة فقط لما 2πr2 = πrh =⇒ 2r = h Benelux2014 3939 ∗N ∈ an, · · · , a2, a1 . جد القيمة الاصغر ية للعدد (n ≥ 3) ليكن ⌊ S − a1 a1 ⌋ + ⌊ S − a2 a2 ⌋ + · · · + ⌊ S − an an ⌋ S = a1 + a2 + · · · + an مع i الحل وضوحا لدينا ⌊ S − ai ai ⌋ = ⌊ S ai ⌋ − 1 240 240


. ومنه نحاول ان نقوم بتصغير g(a1, a2, ..., an) = (∑n i=1 ⌊ S ai ⌋) − n ≡ f .من الواضح ان (∑n i=1 S ai ) ليكن n− g ≤ f < g + n −→ g > f − n لدينا f = (a1 + a2 + ... + an)( 1 a1 + 1 a2 + ... + 1 an ) − n ≥ n 2 − n 2)1 − n ≥ (g −→ 2n − n2 > g لان g عدد طبيعي . لـكي نرى ونتاكد من ان هذا يمكن تحقيقيه فقط نقوم ومنه ومنه. a1 = n, ai = n + 1(i ≥ 2) بوضع g = (∑n i=2 ⌊ n 2 + n − 1 n + 1 ⌋) + ⌊ n 2 + n − 1 n ⌋ − n = (∑n i=2 ⌊ n(n + 1) − 1 n + 1 ⌋) + ⌊ n(n + 1) − 1 n ⌋ − n = (∑n i=2 n − 1 ) + n − n = (n − 1)2 اذن ختاما لدينا . اذا كان a1, a2, · · · , an ∈ N ∗ (n ≥ 3) و S = a1 + a2 + · · · + an ⌊ . فان S − a1 a1 ⌋ + ⌊ S − a2 a2 ⌋ + · · · + ⌊ S − an an ⌋ ≥ (n − 1)2 . JWIMC2019 4040 احسب مايلي π ∫4 − π 4 cos x + 1 − x 2 (1 + x sin x) √ 1 − x 2 dx i الحل اولا ناخذ لمحة بسيطة على المساواة الاتية cos x + 1 x sin x + 1 dx = d(x + sin x) x sin x + 1 241 241


والتي تلهمنا لحساب d dx x + sin x x sin x + 1 = cos x(cos x + 1 − x 2 ) (x sin x + 1)2 . x sin+x := y ,وبالمقارنة مع التكامل لدينا x sin x+1 ليكن I := ∫ cos x + 1 − x 2 (1 + x sin x) √ 1 − x 2 dx = ∫ x sin x + 1 cos x √ 1 − x 2 dy = ∫ x sin x + 1 √ (1 − x 2 ) ( 1 − sin2 x ) dy. من المهم ان نلاحظ ان ( 1 − x 2 ) (1 − sin2 x ) = (x sin x + 1)2 − (x + sin x) 2 . والتي ينتج عنها I = ∫ ( 1 − (x + sin x) 2 (x sin x + 1)2 )− 1 2 dy = ∫ dy √ 1 − y 2 = arcsin y + C. هنا I1 := ∫ π 4 − π 4 cos x + 1 − x 2 (1 + x sin x) √ 1 − x 2 dx = 2 arcsin ( 4 + π √ 2 π + 4√ 2 ) . JWIMC2009 4141 ليكن a ,b ,c اعداد حقيقية موجبة بحيث 1 = c + b + a .برهن ان 3 √( 1 + a b + c )1−a bc ( 1 + b c + a )1−b ca ( 1 + c a + b )1−c ab ≥ 64 i الحل المتراجحة المعطاة تكافئ ∏ cyc ( 1 + a b + c )( 1−a bc ) ≥ 2 18 242 242


⇔ ∑ cyc ( 1 − a bc ) ln ( 1 + a 1 − a ) ≥ 18 ln 2 ليكن f(x) = ln ( 1 + x 1 − x ) الحساب المباشر للمشتقة الاولى والثانية للدالة f يعطي لنا f ′ (x) = − 2 x 2 − 1 و f ′′(x) = 4x (x 2 − 1)2 وبالتالي f محدبة ومتزايدة على المجال (1, 0( باستعمال متراجحة s'Jensen نحصل على ∑ cyc ( 1 − a bc ) ln ( 1 + a 1 − a ) ≥ (∑ cyc 1 − a bc ) ln (∑ cyc 1+a ∑ bc cyc 1−a bc ) ⇒ ∑ cyc ( 1 − a bc ) ln ( 1 + a 1 − a ) ≥ 2 (∑ cyc 1 a ) ln ( 1 + ∑ cyc a 2 1 − ∑ cyc a 2 ) الان باستعمال متراجحة Cauchy shwartz لدينا ( ∑ cyc a 2 )(1 + 1 + 1) ≥ ( ∑ cyc a) 2 = 1 ⇒ ∑ cyc a 2 ≥ 1 3 · · ·(1) وكذلك باستعمال متراجحة GM-AM لدينا (∑ cyc a ) (∑ cyc 1 a ) ≥ 9 ⇒ ∑ cyc 1 a ≥ 9 · · ·(2) الان بجمع كل ماحصلنا عليه واستعمال حقيقة f متزايدة نحصل على ∑ cyc ( 1 − a bc ) ln ( 1 + a 1 − a ) ≥ 18 ln 2 وهو المطلوب برهانه . NMC2021 4242 جد جميع الدوال R− > R : f التي تحقق من اجل كل عدد حقيقي x مايلي f(x)(1 + |f(x)|) ≥ x ≥ f(x(1 + |x|)) 243 243


i الحل = x في −1+√ 1+4t 2 ≥ 0 بوضع t > 0 ليكن x ≥ f(x(1 + |x|) , نحصل على f(t) ≤ −1 + √ 1 + 4t 2 , ∀t ≥ 0 اذا كان 0) ≤ t(f فان f(t)(1 + |f(t)|) ≥ t سيستلزم ان 0 ≤ t ,غير ممكن ومنه 0) > t(f ولدينا 0 < f(t) ≤ −1 + √ 1 + 4t 2 ومنه f(t)(1 + |f(t)|) ≥ t تصبح f(t) 2 + f(t) − t ≥ 0 ومنه f(t) ≥ −1 + √ 1 + 4t 2 ومنه f(x) = −1 + √ 1 + 4x 2 , ∀x > 0 ومنه f(0) = 0, f(x) = |x| 2x (√ 1 + 4|x| − 1 ) ∀x ̸= 0 NMC2017 4343 ≥ β, α ≤ 0 .برهن انه اذا كان π 2 و, 0 ≤ a, b ≤ 1 بحيث حقيقية اعداد a, b, α, β ليكن ab cos(α − β) ≤ √ (1 − a 2 )(1 − b 2 ), فان a cos α + b sin β ≤ 1 + ab sin(β − α). 244 244


i الحل 1 − a 2 − b 2 + a 2 b 2 ≥ a 2 b 2 cos2 (α − β) = a 2 b 2 (1 − sin2 (α − β)) = a 2 b 2 − a 2 b 2 sin2 (α − β)a 2 b 2 sin2 (α − β) ≥ a 2 + b 2 − 1 (1 − ab sin(α − β))2 = 1 + a 2 b 2 sin2 (α − β) − 2ab sin(α − β) ≥ a 2 + b 2 − 2ab sin(α − β)(a cos α + b sin β) 2 = a 2 cos2 α + b 2 sin2 β + 2ab cos α sin β = a 2 + b 2 − a 2 sin2 α − b 2 cos2 β + 2ab cos α sin β = a 2 + b 2 − (a sin α + b cos β) 2 + 2ab sin α cos β + 2ab cos α sin β = a 2 + b 2 − (a sin α + b cos β) 2 + 2ab sin(α + β) = a 2 + b 2 − 2ab sin(α − β) − (a sin α + b cos β) 2 + 2ab sin α cos β = a 2 + b 2 − 2ab sin(α − β) − (a sin α − b cos β) 2 ≤ a 2 + b 2 − 2ab sin(α − β) ≤ (1 − ab sin(α − β))2 IMS2006 4444 برهن انه من اجل كل 1 ≥ m :لدينا ∑ |k|< √ m ( 2m m + k ) ≥ 2 2m−1 i الحل نلاحظ ان ∑ |k|> √ m ( 2m m + k ) = ∑ |k|≥√ m m ( 2m m+k ) m ≤ ∑ |k|≥√ m k 2 ( 2m m+k ) m ≤ ∑m k=−m k 2 ( 2m m+k ) m = m2 2m−1 m = 22m−1 245 245


4545 برهن انه من اجل كل 2 ≥ n تكون المتراجحة الاتية محققة : 1 n + 1 ( 1 + 1 3 + . . . + 1 2n − 1 ) > 1 n ( 1 2 + . . . + 1 2n ) . i الحل 4 , صحيحة . 9 > 3 8 نستعمل البرهان بالتراجع .اذن كشرط ابتدائي من اجل 2 = n لدينا 1 . ومنه , n+1 · ( 1 + 1 3 + · · · + 1 2n−1 ) > 1 n · ( 1 2 + · · · + 1 2n ) الان نفرض ان 1 + 1 3 + · · · + 1 2n − 1 > n 2(2n + 1) 1 (n + 1)2 · (n + 2) · ( 1 + 1 3 + · · · + 1 2n − 1 ) > (2n + 1)(n + 2) − (2n + 2)(n + 1) (n + 1)(n + 2)(2n + 1)(2n + 2) [ 1 n + 2 − n (n + 1)2 ] · ( 1 + 1 3 + · · · + 1 2n − 1 ) > 1 (n + 1)(2n + 2) − 1 (n + 2)(2n + 1) 1 n + 2 · ( 1 + 1 3 + · · · + 1 2n + 1) > n (n + 1)2 · ( 1 + 1 3 + · · · + 1 2n − 1 ) + 1 2 · 1 (n + 1)2 نستعمل فرضية التراجع نحصل على 1 n + 2 · ( 1 + 1 3 + · · · + 1 2n + 1) > n (n + 1)2 · [ n + 1 n · ( 1 2 + 1 4 + · · · + 1 2n )] + 1 2 · 1 (n + 1)2 1 n + 2 · ( 1 + 1 3 + · · · + 1 2n + 1) > 1 n + 1 · ( 1 2 + 1 4 + · · · + 1 2n + 1 2n + 2) ومنه المتراجحة محققة من اجل 1 + n اذا كانت محققة من اجل n وبالتالي التراجع اكتمل استمرار ية دالة الجذر التربيعي عند الصفر 4646 الـكثير من التلاميذ والزملاء الافاضل يتساءلون عن استمرار ية دالة الجذر التربيعي عند الصفر .سنحاول ادناه تقديم الاجابة على هذا السؤال بشكل بسيط حتى يتسنى للجميع فهم استراتيجية هذا المشكل . i الاجابة نقول دالة الجذر التربيعي مستمرة عند الصفر باستعمال البناء .دالة الجذر التربيعي معرفة على المجال (∞ ,0+ = [R .الان من اجل اي متتالية N∈n)xn (في المجال السابق (المتتالية ذات القيم الموجبة 0 ≥ xn من اجل كل N ∈ n (المتقاربة نحو 0 ,اذن يمكن ان يكون لدينا كدالة مرافقة ذات √. والتعر يف الذي طبقناه هنا هو احد التعار يف المكافئة في الفضاءات المترية العامة اما √ متقاربة نحو 0 = 0 القيم xn الحديث عن استمرار ية دالة الجذر التربيعي على يسار الصفر اين تكون الدالة غير معرفة فهذا لايمكن لان الدالة اصلا لاتملك صور عند 0 < x وبالتالي لايمكن الحديث عن موضوع الاستمرار ية هنا . 246 246


AlbanianTST2014 ii حل المعادلة الاتية في R: ( x − 1 x )1 2 + ( 1 − 1 x )1 2 = x. iii الحل (x − 1 x ) 1 2 + (1 − 1 x ) 1 2 = x, x > 0 و 1 − 1 x ≥ 0 ⇒ x ≥ 1 و t = 1 x ∈ (0, 1] , (x − 1 x ) 1 2 + (1 − 1 x ) 1 2 x = ⇐⇒ t 1 2 (1 − t 2 ) 1 2 + t(1 − t) 1 2 = , 1 = t 1 2 (1 − t 2 ) 1 2 + t(1 − t) 1 2 ≤ t + 1 − t 2 2 + t 2 + 1 − t 2 = 1 ⇒ t = 1 − t 2 ⇒ t = 1 − √ 5 2 ⇒ x = 1 + √ 5 2 . ICM2019 4747 b .برهن ان 239 = bc − 1 و a 239 = ac − 1 بحيث موجبة صحيحة اعداد a, b, c ليكن 2382 (ab) 239 < 1 i الحل a 239 + 1 a = b 239 + 1 b ab(a 238 − b 238) − (a − b) = 0 ab(a 237 + a 236b + ... + b 237) = 1 1 = ab(a 237 + ... + b 237) > ab ∗ 238 238√ (ab) 237∗238 2 = 238 ∗ (ab) 239 2 ومنه 2382 (ab) 239 < 1 247 247


4848 متتاليات زراولية الهدف من هذا الجزء هو حل مشكل تقارب متتاليات زراولية وهي عبارة عن متتاليات تلسكوبية الاس معرفة بالشكل الاتي an = (1 − 1 2 ) ( 1 2 − 1 3 ) ... ( 1 n − 1 n+1 ) تقارب هاته المتتالية غير معروف الى حد الساعة قمنا بنشر هذا النوع من المتتاليات في الموسوعة المباشرة للاعداد الصحيحة في اكتوبرعام 2019 يمكن الاطلاع عليهما في الرابط [16 [وبالضبط المتتاليتين A328941 و A328941 سلوك المتتاليتين يتوقف على شفعية العدد الطبيعي n حيث اذا كان n فردي اي باخذ التكرارات فردية تكون المتتالية A328941 متقاربة نحو العدد 0.56778606544394002098000796382530333102219963214865 · · · اما اذا كان زوجيا اي باخذ التكرارات زوجية تكون المتتالية A328942 متقاربة نحو العدد 0.85885772008416606762434379473241623070938618180813 · · · . السؤال المطروح هو " هل ان متتاية زراولية متقاربة اي هل ان limann → ∞ موجودة ? بعض البراهين التي قدمها الباحثين في احد المواقع التفاعلية لحل هذا المشكل ولـكنها تلقت تحدي كبير من هذا المشكل سنحاول استعراض بعض البراهين على النحو الاتي i البرهان الاول من طرف heropup الحساب العددي للمتتالية 1≥n}an {تقترح ان حدود المتتالية محدودة وٺتناوب تقريبيا بين 0.56778606544394002098000796382530333102219963214866 و 0.85885772008416606762434379473241623070938618180813, لـكن لااملك برهان . التقارب هنا جد سر يع والتناوب الطبيعي يقترح علينا انه من الضروري لفت الانتباه الى شفعية العدد الطبيعي n الزوجية والفردية منها على حدا ii البرهان الثاني من طرف Barry Cipra في هذا البرهان فقط نوضح ان نهاية المتتالية المعنونة لايمكن ان تكون 1. .......................................... نلاحظ ان an = (1/2)(1/6)(1/12)··· , مع ان النقاط " · · ·" تعني الانتهاء عند الاس ((1 + n(n/(1 .كقاعدة عامة اذا كان 1 < r < 0 و 1 < b < a < 0, ان ينتج فانه. 0 < r < rb < ra < 1 0 <(1/12) <(1/12)(1/20)··· < 1 248 248


وبالتالي ايضا لدينا 0 <(1/6) <(1/6)(1/12)(1/20)··· <(1/6)(1/12) < 1 ومنه ختاما لدينا 0.5504566141 ≈ (1/2)(1/6)(1/12) <(1/2)(1/6)(1/12)··· <(1/2)(1/6)≈0.89089871814 وهاته القيم الحدية ٺتوافق مع الشيئ الذي حصل عليه heropup. iii البرهان الثالث من طرف Yuri Negometyanov بعض القيم الاولى للمتتالية an = {2 −1 , 2 −6−1 , 2 −6−12−1 , . . . 2 −6−12...−(n(n+1))−1 } معطاة بالشكل الاتي 0.5, 0.890899, 0.550457, 0.867251, 0.56342, 0.860843, 0.566835 . . . من السهل ان نلاحظ ان المتتالية من اجل التكرارات الزوجية والفردية مختلفة ومن جهة اخرى لدينا .اذا كان limn→∞ an موجودة فانها تمثل نهاية كل من المتتاليتين الزوجيتين والفرديتين . ليكن (1) tn = (n(n + 1))−((n+1)(n+2))−((n+2)(n+3))... ومنه (2) tn = (n(n + 1))−tn+1 (3) tn+1 = − log tn log (n(n + 1)) . لنعتبر الان المتتالية ,Tn بحيث (4) limn→∞ Tn = limn→∞ Tn+1 مع Tn تمثل جذر المعادلة الاتية (5) Tn = − log Tn log (n(n + 1)) (6) Tn == e −W(log(n 2+n)) W-function Lambert دالة تمثل W(x) مع 249 249


من السهل ان نلاحظ ان (7) 2 −6 ...−((n−1)n)−Tn = 2−6 ...−((n−1)n)−(n(n+1))−Tn وهذا يعني انه يمكننا تعر يف المتتالية (8) bn = 2−6 ...−((n−1)n)−tn b1 ≈ 2 −e−W(log(6)) , b2 ≈ 2 −6−e−W(log(12)) , b3 ≈ 2 −6−12−e−W(log 20)) . . . , مع ضعف اكثر في الفرق بين المتتاليتين الجزئيتين الفردية والزوجية هذا التقريب يسمح لنا للحصول على التقريب الاكثر استقرارا ل a وتزود الاصدار .1 ̸= a كل قيمة للنهاية الممكنة a تولد المتتالية tn بالصيغة .(3) اذا كانت المتتالية المحصل عليها غير رتيبة فان قيمة a خاطئة اعتبار الحالة ∞ → n تسمح لنا بالحصول على النهايات al و ah للقيمة .a على سبيل المثال القيمة المتتالية تولد ah = 0.719 tn = {0.719, 0.475936, 0.414381, 0.354528, 0.311916, 0.311697, 0.289595, 0.289775, 0.275267}, والتي هي رتيبة .من السهل ان نلاحظ ان هاته المتتاليات مع a > ah ليست رتيبة هذا يسمح لنا بتخمين او تاكيد المتباينة الاتية a < ah < 0.719. بنفس الطر يقة يمكننا ان نبين ان a > al > 0.711, باعتبار tn = {0.711, 0.492079, 0.395766, 0.373025, 0.329171, 0.326702, 0.299306, 0.299673, 0.281777} وبالتالي القيمة الممكنة للنهاية محدودة مع: a ∈ (0.711, 0.719) . في نفس الوقت الحساب العددي في المراحل الثلاثة 250 250


Click to View FlipBook Version